LSAT and Law School Admissions Forum

Get expert LSAT preparation and law school admissions advice from PowerScore Test Preparation.

 Administrator
PowerScore Staff
  • PowerScore Staff
  • Posts: 8917
  • Joined: Feb 02, 2011
|
#26956
Complete Question Explanation
(The complete setup for this game can be found here: lsat/viewtopic.php?t=11291)

The correct answer choice is (E)

The condition in the question stem creates the following chain:
Sept 06_game #3_M12_L4_explanations_game#3_16_diagram_1.png
This chain address all six events involving the friends. Answer choice (C) cannot occur because R bought his car earlier than T bought his car, and hence R cannot have bought his car in 1995. Answer choice (E) is thus correct in this except question.

Get the most out of your LSAT Prep Plus subscription.

Analyze and track your performance with our Testing and Analytics Package.